GYN MIDTERM: CHP. 15-19
Fluid noted anterior to the uterus would most likely be located within the: a. Pouch of Douglas b. Vesicouterine pouch c. Space of Retzius d. Rectouterine pouch
B. Vesicouterine pouch
Pelvic ligament that extends from the lateral surface of the cervix to the lateral fornix of the vagina and houses the uterine vasculature
Cardinal ligament
What ligament extends from the lateral surface of the cervix to the lateral fornix of the vagina and supports the cervix?
Cardinal ligament
What are the sonography findings of a cystic teratoma?
Complex, partially cystic mass in the ovary that includes one or more academic structures that make Shadow,, tip of the iceberg sign only the interior element of the masses seen while the greater part of the mass is obscured by shadowing,, dermoid plug produces posterior shadowing,, dermoid mesh produced by hair and will appear as numerous linear interfaces within the cystic area of this Mass
What are the potential differential gynecologic diagnosis of elevated white blood cell count?
Endometritis, pelvic inflammatory disease
What fibroids usually lead to abnormal uterine bleeding because of their location in relationship to the endometrium?
Intracavitary
What are the sonography appearance of serous cystadenocarcinoma?
Large, multilocular cystic mass, papillary projections and step tations often noted, ascites
What are the sonography findings of endometrioma?
Most often multiple, predominantly cystic mass with low-level internal echoes, May resemble a hemorrhagic cyst, anechoic or complex mostly cystic mass with posterior enhancement and may have a fluid fluid level
What are the potential differential gynecologic diagnosis of precocious puberty?
Ovarian dysgerminoma, ovarian granulosa cell tumor
Peritoneal spaces located posterior to the broad ligament are referred to as the: a. Rectouterine spaces b. Anterior cul-de-sacs c. Lateral cul-de-sacs d. Adnexa
d. adnexa
all of the following are sonographic findings consistent with adenomyosis except: a. diffuse, enlarged uterus b. myometrial cysts c. hypoechoic areas adjacent to the endometrium d. complex adnexal mass
d. complex adnexal mass
the uterus of attachment of the fallopian tubes to the uterus is the: a. fundus b. corpus c. isthmus d. cornua
d. cornua
which of the following would be caused by a large acoustic interface and subsequent production of false echoes? a. posterior shadowing b. acoustic enhancement c. mirror image d. reverberation
d. reverberation
what congenital malformation of the uterus is common and has a clear association with an increases risk for spontaneous abortion? a. anteflexed b. levoverted uterus c. dextroverted uterus d. septate uterus
d. septate uterus
blood within the tube
hematosalpinx
The area within the brain that is located just beneath the hypothalamus that controls and releases the hormones by the anterior pituitary gland
Hypothalamus
A vaginal and Emily in which the hymen has no opening, therefore resulting in an obstruction of the vagina
Imperforate hymen
The term used for the Isthmus of the uterus during pregnancy
Lower uterine segment
The first 4 weeks after birth
Neonatal
What are the potential differential gynecologic diagnosis of elevated serum lactate dehydrogenase?
Ovarian dysgerminoma
- most common benign ovarian tumor - results from retention of unfertilized ovum differentiates into 3 germ cells - complex, partially cystic - bone, hair, fat, cartilage, teeth
cystic/dermoid teratoma
the most superior and widest portion of the uterus is the: a. corpus b. isthmus c. cervix d. fundus
d. fundus
the location of a fibroid within the myometrium is termed: a. submucosal b. intracavitary c. subserosal d. intramural
d. intramural
What day of the menstrual cycle does ovulation occur?
day 14
Around what day is LH produced by the anterior pituitary gland which stimulates ovulation at which time the graafian follicle which is grown ruptures and expels a small amount of fluid and the ovum into the peritoneal?
14
What are the clinical findings of corpus luteum cysts and corpus luteum of pregnancy?
Asymptomatic, pain associated with Hemorrhage and enlargement of cyst, corpus luteum of pregnancy accompanies a pregnancy
What are the clinical findings of endometrioma?
Asymptomatic, pelvic pain, and fertility, dysmenorrhea, menorrhagia, despair Junia, painful bowel movements
The inner mucosal layer of the uterus
Endometrium
What are the sonography findings of ovarian torsion?
Enlarged ovary, enlarged ovary in the presence of multi follicular development, lack of or diminished flow patterns compared with the non affected ovary
The hormone of the anterior pituitary gland that causes the development of multiple follicles on the ovaries
Follicle-stimulating hormone
What are the ovaries stimulated by?
Follicle-stimulating hormone
The functional inner layer of the endometrium that is altered by the hormones of the menstrual cycle
Functional layer
The hormone released by the hypothalamus that stimulates the pituitary gland to release the hormones that regulate the female menstrual cycle
Gonadotropin releasing hormone
What is the typical ovarian flow?
High resistant during menstrual and proliferative phases and low resistance at mid cycle
The Doppler analysis of malignant ovarian masses often reveal what?
Higher diastolic flow velocities because of the abnormal vessels that are created with malignancy, these vessels often lack smooth muscle within their walls and produce a less resistive waveform pattern
How does the normal ovary appear?
Homogeneous with a medium level too low level echogenicity, multiple follicles may be noted during neonatal and pre-pubertal ages
A hormone produced by the trophoblastic cells of the early placenta, may also be used as a tumor marker in non gravid patients and males
Human chorionic gonadotropin
Hormone produced by the trophoblastic cells of the early placenta, may also be used as a tumor marker in non gravid patients and males
Human chorionic gonadotropin
The hormone produced by the trophoblast cells of the early placenta may also be used as a tumor marker and non gravid patients and males
Human chorionic gonadotropin
What are the potential differential gynecologic diagnosis of palpable abdominal Mass?
Leiomyoma, leiomyosarcoma
Which of the following is said to be a *common cause of DUB*?
Polycystic ovary syndrome (PCOS)
which of the following could be described as an infection of the female genital tract that may involve the ovaries, uterus, and/or the fallopian tubes? a. pseudomyxoma peritonei b. pelvic inflammatory disease c. polycystic ovarian disease d. ovarian torsion
b. pelvic inflammatory disease
the layer of the endometrium that is significantly altered as a result of hormonal stimulation during the menstrual cycle is the: a. myometrium b. endometrial cavity c. functional layer d. basal layer
c. functional layer
a 13 year old girl presents to the sonography department with a history of cyclic pain, abdominal swelling, and amenorrhea. sonographically, you visualize an enlarged uterus and a distended vagina that contains anechoic fluid with debris. what is the most likely diagnosis? a. cervical stenosis b. adenomyosis c. endometriosis d. hematocolpos
d. hematocolpos
The left ovarian vein drains directly into the: a. Right renal vein b. Inferior vena cava c. Aorta d. Left renal vein
d. left renal vein
what artifact would be seen posterior to a tooth within a cystic teratoma?
d. shadowing
the breast cancer drug that inhibits the effects of estrogen in the breast is:
d. tamoxifen
which of the following statements is not true concerning transabdominal pelvic imaging? a. transabdominal imaging of the pelvis provides a global view of the entire pelvis b. transabdominal imaging lacks the detail of endovaginal imaging c. obese patients and patients with a retroverted or retroflexed uterus present a unique challenge to the transabdominal technique d. transabdominal imaging is contraindicated for pediatric patients
d. transabdominal imaging is contraindicated for pediatric patients
Ovulation usually occurs on or about day _____ of a 28-day ovarian cycle.
day 14
secondary amenorrhea is characteristically diagnosed in what patients
postmenarchal women who has had at least 12 months w/out menstruation
The corpus luteum that is maintained during an early pregnancy for the purpose of producing estrogen and progesterone
Corpus luteum of pregnancy
What are the potential differential gynecologic diagnosis of constipation or painful bowel movements?
Endometriosis, leiomyoma, leiomyosarcoma, ovarian mucinous cystadenocarcinoma, ovarian mucinous cystadenoma, ovarian serous cystadenocarcinoma, ovarian serous cystadenoma
When the sonographic *three-line sign is present the functional layer* of the endometrium typically appears:
Hypoechoic
A 24-year old female patient presents to the emergency department with severe right lower quadrant pain, nausea and vomiting. The sonographic examination reveals an enlarged ovary with no detectable doppler signal. What is the most likely diagnosis? A. Ovarian cystadenocarcinoma B. Cystic teratoma C. Ovarian torsion D. Endometriosis
C. Ovarian torsion
Which of the following is an estrogen-producing ovarian tumor? A. Cystic teratoma B. Fibroma C. Thecoma D. Endometrioma
C. Thecoma
What are the sonography appearance of mucinous cystadenocarcinoma?
Large, multilocular cystic mass, papillary projections and septations often noted, I could genetic material within the cystic components of the mass,pseudomyxoma peritonei (complex ascites)
What is the most common benign gynecologic tumors in the leading cause of hysterectomy and gynecologic surgery?
Leiomyoma
The second phase of the ovarian cycle days 15 to 28 is termed what phase?
Luteal
What are the potential differential gynecologic diagnosis of vaginal discharge?
Pelvic inflammatory disease
The uterine body tilts backward and comes in contact with the cervix for me an acute angle between body and cervix
Retroflexion
The invasion endometrial tissue into the myometrium of the uterus is referred to as: a. amenorrhea b. endometriosis c. adenomyomatosis d. adenomyosis
d. adenomyosis
which of the following would typically not be associated with amenorrhea? a. Asherman syndrome b. polycystic ovarian disease c. pregnancy d. adenomyosis
d. adenomyosis
What other term is used to describe the space of Retzius? a. Posterior cul-de-sac b. Anterior cul-de-sac c. Murphy pouch d. Retropubic space
d. retropubic space
- May occur if graafian follicle fails to ovulate - 3cm-8cm - anechoic, thin wall
follicular cyst
Denotes appearance of mass when only ant element is seen
"tip of the iceberg"
The *average menstrual cycle lasts*:
28 days
What are the potential differential gynecologic diagnosis of pelvic pressure and or tenderness?
Adenomyosis, endometritis, leiomyoma, leiomyosarcoma, ovarian mucinous cystadenocarcinoma, ovarian mucinous cystadenoma, ovarian serous cystadenocarcinoma, ovarian serous cystadenoma, pediatric Hydrocolpos or hematocolpos, pelvic inflammatory disease
The area located posterior to the broad ligament, adjacent to the uterus, which contains the ovaries and fallopian tubes
Adnexa
What are the clinical findings of granulosa cell tumors?
Adolescent - pseudoprecocious puberty,, Reproductive ages and postmenopausal women will have abnormal vaginal bleeding
Asymptomatic what are the sonography findings of a nabothian cyst?
Anechoic Mass within the cervix, maybe multiple, maybe complex
What are the clinical findings of a krukenberg tumor?
Asymptomatic, possible weight loss, pelvic pain
The pelvic ligament that provides support to the ovary and extends from the ovary to the lateral surface of the uterus is the: a. Cardinal ligament b. Ovarian ligament c. Broad ligament d. Suspensory ligament of the ovary
B) Ovarian ligament
Which of the following are the paired anterior abdominal muscles that extend from the xiphoid process of the sternum to the pubic bone? a. Iliopsoas muscles b. Rectus abdominis muscles c. Obturator interni muscles d. Piriformis muscles
B) rectus abdominis muscles
The sonographic appearance of an ovarian dermoid tumor in which on the anterior elements of the mass can be seen, while the greater part of the mass is obscured by shadowing is consistent with: A. Whirlpool sign B. Tip of the iceberg sign C. Dermoid mesh sign D. Dermoid plug sign
B. Tip of the iceberg sign
The abdominal aorta bifurcates into the: a. Internal iliac arteries b. Common iliac arteries c. Ovarian arteries d. External iliac arteries
B. common iliac arteries.
The corpus luteum that is maintained during early pregnancy for the purpose of producing estrogen and progesterone
Corpus luteum of pregnancy
Which of the following sonographic findings would not increase the likelihood of an ovarian malignancy? A. Septation measuring >3mm in thickness B. Irregular borders C. Solid wall nodule D. Anechoic components with acoustic enhancement
D. Anechoic components with acoustic enhancement
A change in menstrual bleeding patterns related to endocrine abnormalities where hormonal imbalances result in endometrial changes in subsequent abnormal bleeding
Dysfunctional uterine bleeding
*Painful and difficult menstruation* is termed:
Dysmenorrhea
Difficult or painful menstruation
Dysmenorrhea
Painful and difficult menstruation
Dysmenorrhea
The outer germ cell layer of the embryo that develops into the skin, hair, and nails, and other structures
Ectoderm
What structure may be noted on the ovary just *prior to ovulation*?
Graafian follicle
What are the muscles visualized on a sonogram?
Rectus abdominis,iliopsoas, obturator internus, piriformis, levator ani, coccygeus
An ultrasound procedure that uses saline instillation into the endometrial cavity and Fallopian tubes to evaluate for internal abnormalities
Sonohysterography
Extraperitoneal space located between the bladder and symphysis pubis that contains fat
Space of retzius
The arteries within the *functional layer* of the endometrium that are altered by the hormones of the ovary and are shed with menstruation are the:
Spiral arteries
Denotes the sonography tech appearance of a cystic teratoma when only the anterior element of the mass is seen, while the greater part of the mass is obscured by shadowing
Tip of the iceberg sign
Describe the condition of stage 3 of ovarian carcinoma
Tumor involves one or both ovaries with confirmed peritoneal Mets outside of the pelvis and or Regional lymph node involvement
Describe the condition of stage 1 ovarian carcinoma?
Tumor is confined to the ovary
What are the clinical findings of adenomyosis?
Uterine enlargement, dysmenorrhea, menometrorrhagia, pelvic pain, despareunia, multiparous
Following the neonatal period the cervical anterioposterior is equal to or slightly greater than what?
Uterine fundus
A patient presents to the sonography department with a history of uterine prolapsed. Which of the following best describes this disorder? a. A condition that results from the weakening of the pelvic diaphragm muscles and allows for the displacement of the uterus, often through the vagina. b. A congenital anomaly that results in the duplication of the uterus. c. A condition that results in the abnormal invasion of the myometrium through the bladder wall leading to hematuria. d. An abnormality that describes the inversion of the myometrium and endometrium.
a condition that results from the weakening of the pelvic diaphragm muscles and allows for the displacement of the uterus, often through the vagina
which of the following statements would be considered an acceptable disadvantage of endovaginal imaging? a. endovaginal imaging has a limited field of view b. the resolution of endovaginal imaging is reduced compared to transabdominal imaging c. endovaginal imaging is more time consuming than transabdominal imaging d. endovaginal imaging can be performed only by female sonographers
a. endovaginal imaging has a limited field of view
The uterine artery branches off of the: a. Abdominal aorta b. Uterine plexus c. Internal iliac artery d. External iliac artery
c. internal iliac artery
sonongraphic appearance of endometrioma
cystic mass with low level echoes resembling a hemorrhagic cyst
what laboratory value would be most useful to evaluate in a patient with suspected internal hemorrhage?
d. hematocrit
Pelvic bones, when visualized on sonography, will appear: a. Anechoic b. Hypoechoic c. Dark d. Hyperechoic
d. hyperechoic
sonographic appearance of torsed ovary
enlarged ovary, with or without presence of multifollicular development
initiates proliferation and thickening of endo
estrogen
what are the 2 ovarian phases
follicular and luteal
malignant ovarian tumor metastasized from GI tract, smooth walled hypoechoic tumor, commonly bilateral and may be accompanied by ascites
krukenberg tumor
After the graafian follicle ruptures, the remaining structure is termed the:
corpus luteum
Progesterone is produced by
corpus luteum
malignant, less often bilateral, associated with pseudomyxoma
cystadenocarcinoma
The paired embryonic ducts that develop into the female urogenital tract are the: a. fallopian ducts b. wolffian ducts c. gartner ducts d. mullerian ducts
d. mullerian ducts
The muscles that may be confused with the ovaries on a pelvic sonogram include the: a. Rectus abdominis and obturator internus muscles b. Levator ani and coccygeus muscles c. Obturator internus and levator ani muscles d. Piriformis and iliopsoas muscles
d. piriformis and iliopsoas muscles
LH surges on what day? and results in what?
day 14; ovulation
average endo thickness on day 4 and 8
day 4 = 4mm day 8 = 8mm
contains various tissues causing shadowing
dermoid plug
double layer thickness includes what
distance from basal layer to basal layer
patients with granulosa cell tumors have a 10-15% chance of devloping what due to consistent estrogen stimulation
endometrial carcinoma
- benign hemorrhagic tumor that forms from implantation of ectopic endo tissue - functional endo tissue causes hormones to act as it is uterus causing hemorrhage
endometrioma
ovarian tumor that has a high incidence of being malignant. most common cancer to originate within endometrioma. Appears as a complex mass w/ solid components
endometroid tumor
the cells surrounding the follicle produces what
estrogen
as endocrine glands, the ovaries are responsible for producing what hormones
estrogen and progesterone
the ovary produces what two hormones
estrogen and progesterone
what endo layer changes throughout menstruation, location of placenta
functional layer
hormone produced by the hypothalamus
gonadotropin-releasing hormone
- sex cord stromal tumor - most common estrogenic tumor - unilateral - postmenopausal women
granulosa cell tumors
Which hormone *maintains the corpus luteum* during pregnancy?
hCG
follicular cyst that contains blood, appears complex or completely echogenic
hemorrhagic cyst
doppler analysis of malignant ovarian masses often reveal what
high diastolic flow velocities because of abnormal vessels created with less resistance.
typical ovarian flow during menstrual and proliferative phase
high resistance - menstrual low resistance - midcycle
The normal sonographic appearance of the ovary
homogenous with medium to low level echogenicity
blastocyst produce what hormone
human chorionic gonadotropin (hCG)
patients with theca lutein cysts may suffer from
hyperemesis and pelvic fullness
may cause follicular cyst
hyperstimulation of ovary
distal portion of tube that provides an opening to peritoneal cavity
infundibulum
within the cornu of the uterus lies the _________ extension of the fallopian tube
intramural
means "bridge". Short and narrow segment of the tube connecting interstitial area of ampulla
isthmus
pituitary gland is also known as what
master gland
means middle pain, pain at time of ovulation
mittleschmerz
often larger , tend to have septations but not usually bilateral, INTERNAL DEBRIS
mucinous cystadenoma
signs and symptoms of ovarian torsion
nausea, vomiting, abdominal pain
ovarian fossa includes what
outer cortex and inner medulla
three layers of the fallopian tube
outer serosa, middle muscular layer, inner mucosa
- Mass of ovary - Site of oogenesis
ovarian cortex
located posterior to ureter and internal iliac artery. Superior to external iliac artery
ovarian fossa
if corpus luteum cyst become too large it can lead to
ovarian torsion
reoccurs most often on right ovary, most common cause being ovarian cyst or mass
ovarian torsion
paired, oval shaped intraperitoneal organs with a dual blood supply
ovaries
small cysts adjacent to ovary, may have septations, larger may cause torsion
paraovarian cyst
follicular phase of ovarian cycle
period of follicle growth (days 1-14)
patients with dysgerminoma may present with what
precocious puberty and elevate hCG.
infection within the tube often caused by PID
salpingitis
sex cord stromal ovarian neoplasm associated with virilization. May present with hirsutism. Often women <30yrs, may be malignant, may appear as solid hypoechoic masses or complex cystic mass
sertoli - leydig cell tumor
The trophoblastic cells surrounding the Blastocyst that are responsible for producing human chorionic gonadotropin
syncytiotrophoblastic cells
When a Graafian follicle ruptures it forms what
the corpus luteum
the one follicle remaining on the ovary turns into what
the graafian follicle/dominant follicle
All Venus structures mirror their arterial counterparts with the exception of what?
the left ovarian vein drains into the left renal vein
- largest and least common cyst - elevated hCG levels (>100,000) - common with gestation trophoblastic - multiple gestation increases risk
theca lutein cyst
- benign ovarian sex cord stromal tumor - most common in postmenopausal women - estrogen producing tumors - hypoechoic, solid mass w/post attenuation - unilateral
thecoma
The urinary bladder, uterus, and ovaries are located within the: a. True pelvis b. False pelvis
true pelvis
A benign smooth muscle tumor of the uterus AKA fibroid AKA uterine Myoma
Leiomyoma
The malignant manifestation of a leiomyoma
Leiomyosarcoma
The *second phase of the ovarian cycle* is called the:
Luteal phase
Developing into cancer
Malignant degeneration
Ascites and pleural effusion in the presence of some benign ovarian tumors
Meigs syndrome
The *first menstrual cycle* is termed:
Menarche
Abnormally heavy and prolonged menstration
Menorrhagia
Abnormally heavy and prolonged menstruation
Menorrhagia
The germ cell layer of the embryo that develops into the circulatory system, muscles, reproductive system, and other structures
Mesoderm
Irregular menstrual bleeding between periods
Metrorrhagia
Which is larger serous or mucinous cystadenoma?
Mucinous
Which malignant ovarian tumor is often associated with pseudomyxoma peritonei?
Mucinous cystadenocarcinoma
Paired embryonic ducks that develop into the female urogenital tract
Mullerian ducts
Having more than one internal cavity
Multiloculated
Having birth more than one child
Multiparous
The muscular layer of the uterus
Myometrium
Who are at a greater risk for development of fibroids?
Obese, black, non-smokers, perimenopause
The creation of an ovum
Oogenesis
A syndrome resulting from hyperstimulation of the ovaries by fertility drugs, results in the development of multiple, enlarged follicular ovarian cysts
Ovarian hyperstimulation syndrome
What are the potential differential gynecologic diagnosis of oliguria?
Ovarian hyperstimulation syndrome
What are the potential differential gynecologic diagnosis of ovarian enlargement?
Ovarian hyperstimulation syndrome
What are the potential differential gynecologic diagnosis of nausea and vomiting?
Ovarian hyperstimulation syndrome, ovarian torsion
Massive amounts of pelvic ascites may be associated with what?
Ovarian tumors, cirrhosis, portal hypertension
Pseudoprecocious puberty has been linked with what?
Ovarian, adrenal, and liver tumors
Following ovulation the ovary Begins the what phase while the endometrium enters the what phase?
Ovary Begins the luteal phase while the endometrium enters the secretory phase
Fallopian tubes AKA
Oviducts, uterine tubes, salpiges
The release of the mature egg from the ovary
Ovulation
When is Meg syndrome present?
Pelvic ascites is accompanied by pleural effusion and a benign ovarian Mass
Salpingitis is often caused by what?
Pelvic inflammatory disease
What are the clinical findings of leiomyosarcoma
Pelvic pressure, menorrhagia, palpable abdominal Mass, enlarged bulky uterus, urinary frequency, dysuria, constipation, infertility
What are the clinical findings of the uterine leiomyoma?
Pelvic pressure, menorrhagia, palpable pelvic Mass, enlarged bulky uterus, urinary frequency, dysuria, constipation, infertility
Mullerian ducts AKA?
Perimesonephric ducts
What are the potential differential gynecologic diagnosis of hirsutism?
Polycystic ovarian disease, sertoli leydig cell tumor
If a patient has menstrual cycles that are 17 days apart, what would you describe this as?
Polymenorrhea
What is defined as frequent *regular cycles but less than 21 days apart*?
Polymenorrhea
The vagina is positioned where to the urethra?
Posterior
Where is the ovarian fossa located?
Posterior to the ureter and internal iliac artery and superior to the external iliac artery
Endometrial atrophy is a common occurrence in What patients?
Postmenopausal who presented with vaginal bleeding
Failure to experiencing menarche before age 16
Primary amenorrhea
A hormone that prepares the uterus for the pregnancy, maintains pregnancy, and promotes development of the mammary glands primarily produced by the ovaries and placenta
Progesterone
The *corpus luteum primarily releases*:
Progesterone
What maintains the thickness of the endometrium in preparation for implantation?
Progesterone
A condition that results from the weakening of the pelvic diaphragm muscles and allows for displacement of the uterus, often through the vagina
Prolapse
The multiplication of similar forms
Proliferation
The endometrial cycle consists of what two phases?
Proliferative and secretory
The presence of pus within the fallopian tube
Pyosalpinx
A diagnostic imaging modality that uses ionizing radiation for Imaging bones, organs, and some soft tissue structures
Radiography
What type of uterus makes a challenge for transabdominal technique?
Retroverted or retroflexed
Caused by large acoustic interface and subsequent production of false echoes, echogenic region in the anterior aspect of the urinary bladder
Reverberation
Ovarian torsion often occurs on which side? What is the most common cause?
Right, ovarian cyst or mass such as the benign cystic teratoma or paraovarian cyst
What ligaments extends from the uterine cornua to labia majora between the folds of the broad ligament and support the uterine fundus?
Round ligament
Characteristically diagnosed in the postmenarcheal woman who has had at least 12 months without a menstrual cycle
Secondary amenorrhea
What phase of the endometrial cycle occurs after ovulation and is stimulated by progesterone?
Secretory
When the ovary is in the *luteal phase*, the endometrium is in the:
Secretory
The *second phase* of the endometrial cycle is the:
Secretory phase
The second phase of the endometrial cycle
Secretory phase
Congenital malformation of the uterus that results in a single septum that separates two endometrial cavities
Septate uterus
A partition separating two or more cavities
Septations
During the late proliferative phase or Periovulatory phase endometrial layer displays what?
Stark contrast and can measure between 6 and 10 mm the outer academic basal layer surrounds the more hypoechoic functional layer while the functional layer is separated by the echogenic endometrial stripe AKA three lines sign
What is the primary role of gonadotropin-releasing hormone?
Stimulates release of hormones by the anterior pituitary gland.
Krukenberg tumor is a malignant ovarian tumor that metastasized from the GI tract most frequently the what?
Stomach
A leiomyoma that distorts the shape of the endometrium
Submucosal
Congenital malformation of the uterus that results in abnormal uterine Contour with an endometrium that branches into two horns
Subseptate uterus
What ligament extends from the ovaries to the pelvic sidewalls and supports ovaries and tubes?
Suspensory ligament of the ovaries, infundibulopelvic
Pelvic ligament that provides support to the ovary and extends from the ovaries to the pelvic sidewalls
Suspensory ligament of the ovary
A breast cancer drug that inhibits the effects of estrogen in the breast
Tamoxifen
What happens to the corpus luteum if there is no implantation?
The corpus luteum becomes the corpus albicans due to a decrease in estrogen
Which of the following is a true statement with respect to fallopian tubes?
The fallopian tube is usually only identified if there is an obstruction and it is distended with fluid.
What signals the corpus luteum to continue to produce progesterone when a fertilized egg is approaching the endometrium?
The syncytiotrophoblast cells produce hCG which keeps the corpus luteum producing progesterone.
Cells of the follicle that produce estrogen
Theca internal cells
A 60-year-old female patient presents with elevated estrogen levels, ovarian tumor, ascites, and pleural effusion. What is the most likely diagnosis for the ovarian tumor?
Thecoma
Which of the following is false with respect to theca lutein cysts?
They are found when hCG levels are decreased.
Congenital malformation of the uterus that results in the uterus with one horn
Unicornuate uterus
What are the sonography findings of a Brenner tumor?
Unilateral, small, solid, hypoechoic Mass, may contain calcifications
Having only one internal cavity
Unilocular
Branches of the internal iliac artery that supplies blood to the uterus, ovaries, and fallopian tubes
Uterine arteries
Besides a hysterectomy or myomectomy what is an alternative treatment for fibroids?
Uterine artery embolization
Procedure used to block the blood supply to a leiomyoma
Uterine artery embolization
What ligament extends from the uterus to the sacrum and supports the uterus?
Uterosacral ligament
a 38 year old female patient presents to the sonography department for a pelvic sonogram with an indication of pelvic pain. upon sonography interrogation, the sonographer notes an anechoic mass within the cervix. this is most likely represents a: a. nabothian cyst b. benign follicular cyst c. dermoid cyst d. gartner duct cyst
a. nabothian cyst
which of the following fibroid locations would most likely result in abnormal uterine bleeding because of its relationship to the endometrium? a. submucosal b. intramural c. subserosal d. subserosal pedunculated
a. submucosal
may be caused by number of complications and defined as a change in menstrual bleeding
abnormal uterine bleeding
rare cancer of the fallopian tubes
adenocarcinoma
The longest and most tortuous segment of the Fallopian tube is the. Location of fertilization
ampulla
endometrioma can be located where
anywhere outside of the endometrial cavity
The right ovarian artery branches off of the: a. Aorta b. Right renal artery c. Uterine artery d. Internal iliac artery
aorta
a 24 year old female patient presents to the sonography department for a pelvic sonogram with an indication of pelvic pain. upon sonographic interrogation, the sonographer notes an anechoic mass within the vagina. this mass most likely represents a. nabothian cyst b. gartner duct cyst c. dandy walker cyst d. ovarian cyst
b. gartner duct cyst
A simple fluid accumulation within the vagina secondary to an imperforate hymen is: a. hydrometrocolpos b. hydrocolpos c. hematometra d. hematocolpos
b. hydrocolpos
if fertilization does not occur what happens to the corpus luteum
becomes corpus albicans
the recesses of the vagina are the: a. cornu b. isthmi c. fornices d. parity
c. fornices
another name for endometrioma
chocolate cyst
primary role of the fimbria
draw unfertilized egg into tube
most common malignant germ cell tumor of ovary. Patients <30
dysgerminoma
contains ovarian vasculature and lymphatics
medulla
first menstrual cycle is termed
menarche
days 1 through 5 of menstrual cycle correlate with what
menses
corpus luteum produces what during the luteal phase
progesterone
maintains thickness of endo for implantation
progesterone
what are the 2 uterine phases
proliferative and secretory
the fallopian tube may become distended when
secondary to obstruction or infection
two types of cystadenoma
serous and mucinous
all of the following are common indications for a pelvic sonogram except: a. evaluation of congenital anomalies b. evaluation of pelvic anatomy immediately following a motor vehicle accident c. localization of an intrauterine contraceptive device d. postmenopausal bleeding
!!! b. evaluation of pelvic anatomy immediately following a motor vehicle accident
all of the following would be relevant laboratory tests to evaluate before performing a routine pelvic sonogram except: a. human chorionic gonadotropin b. hematocrit c. white blood cell count d. lipase
!!! d. lipase
The area located posterior to the broad ligament, adjacent to the uterus, which contains ovaries and fallopian tubes
Adnexa
What are the sonography findings of hydrosalpinx?
Anechoic
What are double folds of the peritoneum?
Broad ligament, suspensory ligament of the ovary
Areas just inferior to the fundus of the uterus where the Fallopian tubes attached bilaterally
Cornua
The uterine body
Corpus
The basal layer is slightly altered during the menstrual cycle and consist of what?
Dense cellular stroma
may be idiopathic, related to hormonal imbalances resulting in endo changes. Ex: PCOS
Dysfunctional uterine bleeding
The germ cell layer of the embryo that develops into the GI and respiratory tracts
Endoderm
What are the potential differential gynecologic diagnosis of postmenopausal vaginal bleeding?
Endometrial a trophy, endometrial hyperplasia, ovarian granulosa cell tumor, ovarian mucinous cystadenocarcinoma, ovarian mucinous cystadenoma, ovarian serous cystadenocarcinoma, ovarian serous cystadenoma, ovarian thecoma
Fluid accumulation within the vagina
Hydrocolpos
The abnormal accumulation of fluid within the fallopian tube
Hydrosalpinx
A radiographic procedure that uses a Dye instilled into the endometrial cavity and the Fallopian tubes to evaluate for internal abnormalities
Hysterosalpingography
Location of leiomyoma within the myometrium of the uterus
Intramural
An enzyme found within the blood that may be used to monitor renal function, may also be used as a tumor marker for some ovarian tumors
Lactate dehydrogenase
Destruction or breakdown
Lysis
Pain at the time of ovulation
Middleschmertz
Produced by a strong reflector and results in a copy of the anatomy being placed deeper than the correct location
Mirror image
A patient presents with mittelschmerz. Sonography demonstrated a small amount of free fluid in the right adnexa. What is the likely cause
Ovulation
A small protrusion of tissue
Papillary projections
leiomyoma that extends from the uterus on a stock
Pedunculated uterine leiomyoma
Group of pelvic muscles consisting of the levator Ani and coccygeus muscles that provide support to the pelvic organs
Pelvic diaphragm
Granulosa cell tumors are most commonly found in whom?
Postmenopausal but could be found in younger patients
Defined as pubertal development for the age of eight, the early development of pubic hair, breasts, or genitals
Precocious puberty
the pubertal development before the age of 8, the early development of pubic hair, breasts, or genitals
Precocious puberty
What *hormone maintains the thickness* of the endometrium after ovulation?
Progesterone
Which of the following *hormones is released by the ovary* during the *second half* of the menstrual cycle?
Progesterone
What phase of the endometrial cycle occurs after menstruation and lasts until ovulation?
Proliferative
The first phase of the *endometrial cycle* is the:
Proliferative phase
The first phase of the endometrial cycle
Proliferative phase
What is the primary function of the uterus?
Provide a place for the products of conception to implant and develop
What is the primary purpose of the fallopian tube?
Provide an area for fertilization and to offer a means of transportation for the products of conception to reach the uterine cavity
Secondary sexual development induced by sex steroids or from other sources like ovarian tumors, adrenal tumors or steroid use
Pseudoprecocious puberty
Secondary sexual development induced by sex steroids or from other sources like ovarian tumors, adrenal tumors, or steroid use
Pseudoprecocious puberty
Arteries that Supply blood to the deeper layers of the myometrium
Radial arteries
How is a yolk Sac tumor characterized?
Rapid growth
What are the sonography findings of leiomyosarcoma?
Rapidly growing Mass within the uterus, hypoechoic Mass within the uterus, posterior shadowing from the mass, degenerating fibroids may have calcifications or cystic components, multiple fibroids appear has an enlarged irregular shaped diffusely heterogeneous uterus
Peritoneal outpouching located between the uterus and rectum AKA posterior cul-de-sac and pouch of Douglas
Rectouterine pouch
What pelvic muscles are anterior?
Rectus abdominis
What is the responsibility of the hypothalamus?
Regulate and release of hormones by the anterior pituitary gland the hypothalamus achieves this function by releasing its own hormone gonadotropin-releasing hormone which in turn stimulates the release of hormones by the anterior pituitary gland
What are the ovaries responsible for?
Releasing estrogen and progesterone throughout the menstrual cycle
Endometrioma are found in whom?
Reproductive years
A sonogram of a postpubertal woman demonstrated a 5-cm simple cyst on the left ovary. A follow-up exam 3 months later demonstrates a normal ovary. What do these findings likely represent?
Resolution of a follicular retention cyst
What do dermoids result from?
Retention of an unfertilized ovum that differentiates into the three germ cell layers
The uterine body tilts backward without a Bend where the cervix and body meet
Retroversion
The posterior border of the pelvic cavity is marked by what?
Sacrum and coccyx
What bones Mark the boundaries of the pelvic cavity?
Sacrum, coccyx, and innominate bones
The Bony pelvis consists of what?
Sacrum, coccyx, innominate bones
Tumor marker that is elevated in the presence of an ovarian dysgerminoma and other abdominal abnormalities
Serum lactate dehydrogenase
Caused by attenuation of the sound beam, seen posterior to the dense structures like pelvic bones or teeth within a dermoid
Shadowing
These 1 through 5 of the menstrual cycle correlate with Menses at which this time the endometrium is what?
Shed
What are the sonography findings of a follicular cyst?
Simple cyst - anechoic, thin walled, unilocular, round, posterior enhancement Hemorrhagic cyst - variable appearances including complex components or entirely echogenic depending on the amount of blood in the stage of lysis
What is the sonography appearance of Fallopian tube carcinoma?
Solid Mass within the adnexa, tube may become distended secondary to obstruction or infection, fluid contained within the distended tubes could be simple serous fluid, blood, or pus
Occlusion or in perforation of the vagina, can be congenital or acquired
Vaginal atresia
With congenital malformation of the vagina the obstructions can be the result of what?
Vaginal atresia, vaginal septum, imperforate hymen
Recesses of the vagina
Vaginal fornices
Peritoneal outpouching located between the bladder in the uterus AKA anterior cul-de- sac
Vesicouterine pouch
Female changes within the female that are caused by increased androgens, may lead to deepening of the voice and hirsutism
Virilization
What are the clinical findings of sertoli leydig cell tumors?
Virilization, abnormal menstruation, hirsutism
Do dermoids have the capability of malignant degeneration?
Yes
Ovary malignant germ cell tumor of the ovary
Yolk Sac tumor
Sertoli leydig tumors are found more often in what women?
Younger than 30
Dysgerminoma arise more often in whom?
Younger than 30 and may be found in pregnancy
a patient presents to the sonography department for a pelvic sonogram with a history of adenomyosis that was diagnosed following an MRI of the pelvis. what are the most likely sonographic findings? a. complex, bilateral adnexal masses b. myometrial cysts with enlargement of the posterior uterine wall c. endometrial thinning and cervical dilation d. uterine atrophy with bilateral ovarian cysts
b. myometrial cysts with enlargement of the posterior uterine wall
Both the straight and spiral arteries are branches of the: a. Common iliac artery b. Radial artery c. Arcuate artery d. External iliac artery
b. radial artery
which of the following would be most indicative of a leiomyosarcoma? a. vaginal bleeding b. rapid growth c. dysuria d. large hypoechoic mass
b. rapid growth
The uterine arteries supply blood to all of the following except: a. Fallopian tubes b. Rectum c. Ovaries d. Uterus
b. rectum
The sonographic pelvic examination of a female patient reveals an extensive amount of ascites. In the transverse plane, you visualize two echogenic structures extending from the side walls of uterus to the pelvic side walls bilaterally. These structures are most likely the: a. Broad ligaments b. Cardinal ligaments c. Ovarian ligaments d. Uterosacral ligaments
broad ligaments
all of the following are proper techniques for providing patient care for patients during a pelvic sonogram except: a. all transducers and their cords should be cleaned before performing a pelvic sonogram b. endovaginal transducers should be cleaned with a high-level disinfectant c. a probe cover should be placed on the transducer for transabdominal imaging to prevent the spread of infection d. sterile jelly should be used as a lubricant for endovaginal imaging
c. a probe cover should be placed on the transducer for transabdominal imaging to prevent the spread of infection
abscense of menstruation is referred to as: a. dysuria b. dysmenorrhea c. amenorrhea d. menorrhagia
c. amenorrhea
the normal position of the uterus is: a. retroverted b. retroflexed c. anteverted d. dysverted
c. anteverted
the rigid region of the uterus located between the vagina and the isthmus is the: a. cornu b. corpus c. cervix d. fundus
c. cervix
difficult or painful intercourse is referred to as: a. dysuria b. dysmenorrhea c. dyspareunia d. hydrpcolpos
c. dyspareunia
which of the following is defined as pain during intercourse?
c. dyspareunia
The vagina is located __ to the uterus: a. anterior b. posterior c. inferior d. medial
c. inferior
what abnormality results from the ovary twisting on its mesenteric connection?
c. ovarian torsion
Another name for the rectouterine pouch is the: a. Space of Retzius b. Pouch of Retzius c. Pouch of Douglas d. Anterior cul-de-sac
c. pouch of douglas
What is considered the most dependent part of the peritoneal cavity? a. Space of Retzius b. Anterior cul-de-sac c. Pouch of Douglas d. Rectovessicular pouch
c. pouch of douglas
the "s" in the STAR criteria stand for:
c. smooth walls
The ligament that houses the vasculature of the uterus is the: a. Cardinal ligament b. Ovarian ligament c. Broad ligament d. Suspensory ligament of the ovary
cardinal ligament
the outer layer of the endometrium is the: a. myometrium b. endometrial cavity c. functional layer d. basal layer
d. basal layer
hemorrhage from endometriosis forms into focal areas of bloody tumors called
endometrioma
simple serous fluid within the tube
hydrosalpinx
area within the brain beneath the thalamus
hypothalamus
second phase of the ovarian cycle
luteal phase, days 15 - 28
caused by congenital abnormalities or congenital obstructions
primary amenorrhea
pus in tube
pyosalpinx
dermoids are commonly found in what patients
reproductive age and postmenopausal
cells surrounding blastocyst
syncytotrophoblast
second most common malignant germ cell tumor. Characterized by rapid growth. Female <20. Highly malignant. Elevated AFP.
yolk sac tumor
Fertilization typically occurs on what day?
15
the average menstrual cycle lasts how long
28 days
Approximately what percent of serous cystadenoma are benign occurring more often in women in their 40s and 50s as well as during pregnancy?
50 to 70
How long are the Fallopian tubes?
7 to 12 centimeter
Which of the following best describes the normal appearance of a normal ovary?
<15 mL volume, hypoechoic to myometrium with multiple anechoic follicles
The cystic mass commonly noted with pregnancy is the: A. Corpus luteum B. Dermoid cyst C. Dysgerminoma D. Serous cystadenoma
A. Corpus luteum
The dominant follicle prior to ovulation is termed the: A. Graafian follicle B. Corpus albicans C. Corpus luteum D. Medulla
A. Graafian follicle
Paraovarian cysts are small cysts located where?
Adjacent to ovary
The uterine body tilts forward and comes in contact with a cervix forming an acute angle between body and cervix
Anteflexion
What are the clinical findings of paraovarian cyst?
Asymptomatic, if cyst is enlarged patient may present with pelvic pain and increased lower abdominal girth
Blood within the Fallopian tube is termed: A. Hydrosalpinx B. Hematosalpinx C. Pyosalpinx D. Hemosalpinx
B. Hematosalpinx
The malignant ovarian tumor with gastrointestinal origin is the: A. Brenner tumor B. Krukenberg tumor C. Yolk sac tumor D. Granulosa cell tumor
B. Krukenberg tumor
The ovary is supplied blood by the: a. Ovarian artery b. Ovarian artery and uterine artery c. Uterine artery d. Arcuate artery
B. Ovarian artery and uterine artery
All of the following adnexal masses may appear monographically similar to a uterine leiomyoma except: A. Thecoma B. Paraovarian cyst C. Fibroma D. Granulosa cell tumor
B. Paraovarian cyst
Pelvic ligament that extends from the lateral aspect of the uterus to the side walls of the pelvis
Broad ligament
What ovarian mass is associated with virilization? A. Krukenburg tumor B. Cystic teratoma C. Serous cystadenoma D. Sertoli-Leydig cell tumor
D. Sertoli-Leydig cell tumor
The pelvic ligament that provides support to the ovary to the pelvic side wall is the: a. Cardinal ligament b. Ovarian ligament c. Broad ligament d. Suspensory ligament of the ovary
D. Suspensory ligament of the ovary
What does the cumulus oophorus appear like?
Daughter cyst
The consequence of obstructions for congenital malformation of the vagina could lead to what?
Distention of the vagina, cervix, uterus and Fallopian tubes with blood or fluid
Dysfunctional uterine bleeding is usually related to what?
Endocrine abnormalities in which hormonal imbalances result in endometrial changes and subsequent abnormal bleeding... fibroid tumors, adenomyosis, endometrial hyperplasia, endometrial polyps, endometrial carcinoma
What are the potential differential gynecologic diagnosis of right upper quadrant pain?
Fitz-hugh-curtis syndrome
The name for the dominant follicle prior to ovulation
Graafian follicle
Excessive vomiting
Hyperemesis
The surgical removal of the uterus
Hysterectomy
Tube, the segment of the fallopian tube that is located between the interstitial and ampulla, uterus area of the uterus between the Corpus and the cervix
Isthmus
What happens to the dominant follicle during the luteal phase?
It becomes the corpus luteum, produces progesterone, preparing the endometrium for implantation.
The *hormone that surges* at ovulation is:
LH
What are the sonography findings of theca lutein cyst?
Large bilateral multiloculated ovarian cystic masses, may contain hemorrhagic components
The *periovulatory phase may also be referred* to as the:
Late proliferative phase
Fibromas are a benign ovarian mass that may be Complicated by a condition known as what?
Meigs syndrome
The first menstrual cycle
Menarche
Which of the following could also be described as *intermenstrual bleeding*?
Metrorrhagia
The surgical removal of a Myoma of the uterus
Myomectomy
Hydrocolpos is often seen in what age of patient?
Neonatal
What ligament extends from the ovary to the lateral surface of the uterus and supports ovaries?
Ovarian ligaments
What are the potential differential gynecologic diagnosis of elevated serum Alpha fetoprotein?
Ovarian yolk sac
An infection of the female genital tract that may involve the ovaries, uterus, fallopian tubes
Pelvic inflammatory disease
What are the clinical findings of vaginal obstructions?
Pelvic pain, enlarged uterus, abdominal pain, urinary retention, amenorrhea
What are the clinical findings of a mucinous cystadenoma?
Pelvic pressure and swelling
Another name for the late proliferative phase of the endometrial cycle which occurs around the time of ovulation
Periovulatory phase
What muscles can be confused with the ovaries or adnexal Mass?
Piriformis muscle or iliopsoas
Paired pelvic muscles located posteriorly that extends from the sacrum to the femoral greater trochanter
Piriformis muscles
The abnormal accumulation of fluid in the pleural space
Pleural effusion
Malignant sex cord stromal ovarian neoplasm that is associated with virilization
Sertoli leydig cell tumor
What are the sonography findings of a paraovarian cyst?
Simple cyst located adjacent been out of touch to the ovary, if hemorrhagic will appear complex
Each fallopian tube attaches to the uterus at the level of what?
Uterine horns called the cornua
What is the second most common malignant germ cell tumor?
Yolk Sac tumor
The arteries that directly supply blood to the functional layer of the endometrium are the: a. Radial arteries b. Spiral arteries c. Straight arteries d. Arcuate arteries
b. spiral arteries
The pelvic ligament that extends from the lateral aspect of the uterus to the side walls of the pelvis is the: a. Broad ligament b. Ovarian ligament c. Piriformis ligament d. Round ligament
broad ligament
the inferior portion of the cervix closest to the vagina is the: a. cornu b. internal os c. external os d. inferior fornix
c. external os
the superior portion of the cervix is the: a. cornu b. corpus c. internal os d. external os
c. internal os
which of the following would be the least likely to cause abdominal distension? a. ascites b. multiple leiomyoma c. ovarian hyperstimulation syndrome d. polycystic ovarian disease
!!! d. polycystic ovarian disease
What day is the menstrual cycle correlate with Menses at which time the endometrium is shed?
1 through 5
In secretory phase what is the endometrial thickness?
7 through 14 mm
The most common benign ovarian tumor is the: A. Cystic teratoma B. Mucinous cystadenoma C. Fibroma D. Sertoli- Leydig cell tumor
A. Cystic teratoma
The ovarian mass that contains fat, sebum, and teeth is the: A. Dermoid B. Fibroma C. Mucinous cystadenoma D. Yolk sac tumor
A. Dermoid
The ovarian tumor associated with an elevated serum lactate dehydrogenase is the: A. Dysgerminoma B. Sertoli-Leydig cell tumor C. Androblastoma D. Mucinous cystadenocarcinoma
A. Dysgerminoma
Which of the following is associated with the "whirlpool sign"? A. Ovarian torsion B. Hydrosalpinx C. Ovarian hyperstimulation syndrome D. Ovarian carcinoma
A. Ovarian torsion
A change in menstrual bleeding patterns due to either endocrine abnormalities or lesions within the uterus
Abnormal uterine bleeding
Cancer of the fallopian tube is rare and typically in the form of what?
Adenocarcinoma
The benign invasion of endometrial tissue into the myometrium of the uterus
Adenomyosis
What are the potential differential gynecologic diagnosis of menometrorrhagia?
Adenomyosis, endometrial polyp, pelvic inflammatory disease, perforated intrauterine device
What are the potential differential gynecologic diagnosis of dysmenorrhea?
Adenomyosis, endometriosis
What is the potential differential gynecologic diagnosis for chronic pelvic pain?
Adenomyosis, endometriosis, leiomyoma, pelvic inflammatory disease
What are the potential differential gynecologic diagnosis of dyspareunia?
Adenomyosis, endometriosis, pelvic inflammatory disease
What are the sonography findings of a Gartner duct cyst?
Anechoic Mass within the vagina
Normal ovarian flow is said to be: A. Low resistant during menstruation and high resistant during the proliferative phase B. High resistant during menstruation and low resistant at the time of ovulation C. Low resistant D. High resistant
B. High resistant during menstruation and low resistant at the time of ovulation
Prolapse of the pelvic organs most often involves the: a. Rectus abdominis and obturator internus muscles b. Levator ani and coccygeus muscles c. Obturator internus and levator ani muscles d. Piriformis and iliopsoas muscles
B. Levator ani and coccygeus muscles
The inner layer of the wall of the Fallopian tube is the: A.Muscular layer B. Mucosal layer C. Myometrial layer D. Serosal layer
B. Mucosal layer
The most common uterine anomaly in which the endometrium divides into two horns AKA bicornis unicollis
Bicornuate uterus
The stage at which the conceptus implants within the decidualized endometrium
Blastocyst
Is the involvement of adenomyosis focal or diffuse?
Both, more often within the posterior portion of the uterus
Physical defects that are present in a person at Birth
Congenital malformation
The peripheral arteries of the uterus are the: a. Radial arteries b. Spiral arteries c. Straight arteries d. Arcuate arteries
D) Arcuate arteries
What is the most common malignant germ cell tumor of the ovary?
Dysgerminoma
With menarche what may be influenced?
Environment and diet
Ovarian cyst that forms from the failure of the graafian follicle to ovulate
Follicular cyst
Blood accumulation within the uterine cavity
Hematometra
Blood accumulation within the uterus and vagina
Hematometrocolpos
A cyst that contains blood
Hemorrhagic cyst
What is the most common location for fibroids?
Intramural
What does the term metrorrhagia refer to?
Irregular menstrual bleeding between periods
The innominate bones of the pelvis consist of the: a. Ischium, ilium, and pubic bones b. Ilium, sacrum, and coccyx bones c. Sacrum, coccyx, and pubic bones d. Sacrum, ischium, and ilium bones
Ischium, ilium, and pubic bones
What happens to the corpus luteum if a pregnancy occurs?
It can reach a size of up to 10 cm but should resolve by 16 weeks of gestation.
Destruction or breaking down
Lysis
Posterior cul-de-sac
Male: between the urinary bladder and rectum; also referred to as the rectovesical pouch. Female: between the uterus and rectum; also referred to as pouch of Douglas and rectouterine pouch
What are the clinical findings of theca lutein cyst?
Markedly elevated levels of human chorionic gonadotropin as seen in cases of gestational trophoblastic disease, ovarian hyperstimulation, twin just stations,, nausea and vomiting, pain associated with hemorrhagic, rupture, and ovarian torsion
What are the clinical findings of a thecoma?
May be asymptomatic, postmenopausal vaginal bleeding or abnormal vaginal bleeding secondary to estrogen stimulation, make syndrome which is ascites and pleural effusion
may complicate fibroma with ascites and pleural effusion
Meig Syndrome
Ascites and pleural effusion in the presence of a benign ovarian tumor
Meigs syndrome
What is the definition of primary amenorrhea?
Menarche has not occurred by age 16 in a female patient.
Benign cyst located within the cervix
Nabothian cysts
A mass of tissue that contains abnormal cells AKA tumor
Neoplasm
What pelvic muscles are lateral to the ovaries?
Obturator internus
Pelvic ligament that provides support to the ovary extends from the ovary to the lateral surface of the uterus
Ovarian ligament
On Days 1 through 14 what phases are the ovary in endometrium in?
Ovary is in follicular and endometrium is in proliferative
What pelvic muscles are inferior near the vagina in transverse?
Pelvic diaphragm which includes levator Ani, coccygeus muscle
Infection of the female genital tract that may involve the ovaries, uterus, and or the fallopian tubes
Pelvic inflammatory disease
The outer layer of the uterus AKA serosal layer
Perimetrium
A syndrome characterized by anovulatory cycles, fertility, hirsutism, amenorrhea, and obesity AKA Stein Leventhal syndrome
Polycystic ovarian syndrome
An intraperitoneal extension of mucin secreting cells that result from the rupture of a malignant mucinous ovarian tumor or possibly a malignant tumor of the appendix
Pseudomyxoma peritonei
Intraperitoneal extension of mucin secreting cells that results from the rupture of a malignant mucinous ovarian tumor or possibly a malignant tumor of the appendix
Pseudomyxoma peritonei
An oily substance secreted by the sebaceous glands
Sebum
The outermost layer of the uterus AKA parimetrium
Serosal layer
What is the most common malignancy of the ovary?
Serous cystadenocarcinoma
25-year-old patient presented with hirsutism and abnormal periods. Sonography demonstrated a solid, complex ovarian mass. What is the most likely diagnosis?
Sertoli-Leydig cell tumor
What are the sonography findings of corpus luteum cysts and corpus luteum of pregnancy?
Simple cyst appearance however corpus luteum cyst may have a sticker wall and may be difficult to differentiate from other solid and cystic adnexal masses Hemorrhagic cyst variable appearances including complex components or entirely academic depending on the amount of blood and stage of lysis
What are the sonography findings of sertoli leydig cell tumors?
Solid, hypoechoic ovarian Mass, complex or partially cystic mass
retropubic space
Space of Retzius; between bladder and symphysis pubis
Functional ovarian cysts that are found in the presence of elevated levels of human chorionic gonadotropin, AKA theca luteal cyst
Theca lutein cysts
What are the largest and least common of the functional cysts?
Theca lutein cysts
A periovulatory endometrial sonography appearance in which the outer echogenic basal layer surrounds the more hypoechoic functional layer while the functional layer is separated by the echogenic endometrial stripe
Three line sign
What is the sonographic appearance of the proliferative endometrium?
Three-line sign (echogenic basal layer - hypoechoic functional layer - echogenic canal - hypoechoic functional layer - echogenic basal layer)
Inferior portion of the pelvis that contains the uterus, ovaries, Fallopian tubes, urinary bladder, small bell, sigmoid colon, rectum
True pelvis
Inferior portion of the pelvis that contains the uterus, ovaries, Fallopian tubes, urinary bladder, small bowel, sigmoid colon, and rectum
True pelvis
Describe the condition of stage 2 of ovarian carcinoma
Tumor involves one or both ovaries with pelvic extension
What structure produces *hormones that directly act upon the endometrium* to produce varying thicknesses and sonographic appearances?
Uterus
Congenital malformation of the uterus that results in complete duplication of the uterus, cervix, vagina
Uterus didelphys
The uterine arteries Supply blood to where?
Uterus, Fallopian tubes, ovaries
What develop from the paired mullerian ducts?
Uterus, vagina, fallopian tubes
Yolk Sac tumor is often seen in whom?
Younger than 20 with high malignancy, poor prognosis
which of the following is typically not a clinical complaint of women who are suffering from adenomyosis? a. amenorrhea b. dysmenorrhea c. dyspareunia d. menometrorrhagia
a. amenorrhea (heavy periods)
the uterine position in which the corpus tilts forward and comes in contact with the cervix describes: a. anteflexion b. anteversion c. retroflexion d. retroversion
a. anteflexion
the largest part of the uterus is the: a. corpus b. isthmus c. cervix d. fundus
a. corpus
the surgical removal of a fibroid is termed: a. hysterosonogram b. total abdominal hysterectomy c. myomectomy d. uterine artery embolization
c. myomectomy
leiomyosarcoma of the uterus denotes: a. the benign invasion of endometrial tissue into the myometrium b. the ectopic location of endometrial tissue into the adnexa c. the malignant counterpart of a fibroid d. an anechoic, simple cyst located within the cervix
c. the malignant counterpart of a fibroid
congenital malformation of the uterus that results in complete duplication of the genital tract is: a. unicornuate uterus b. bicornis bicollis c. uterus didelphys d. subseptate uterus
c. uterus didelphys
when does the Centers for Disease Control recommend that alcoho-based handrub not be used by the sonographer?
c. when your hands are visibly soiled
after ovulation, what forms on the ovary and produces progesterone
corpus luteum
develops after ovulation from ruptured follicle, produces progesterone maintaining endo
corpus luteum cyst
50 to 70% benign asymptomatic large and bilateral anechoic lesion with septations or papillary projections
cystadenoma
A drug administered to pregnant women from the 1940s to the 1970s to treat threatened abortions and premature labor that has been linked with uterine malformation in the exposed fetus
do ethyl stop best too DES
what happens when ovulation occurs
dominant follicle ruptures releasing ovum and follicular fluid
first half of menstrual cycle
follicular phase
fingerlike projections form infundibulum
frimbria
The paired muscles that are located lateral to the uterus and anterior to the iliac crest are the: a. Iliopsoas muscles b. Rectus abdominis muscles c. Obturator interni muscles d. Piriformis muscles
iliopsoas muscles
primary purpose of fallopian tubes
provide area of fertilization and to offer a means of transportation for product of conceptus
Which vessels supply blood to the deeper layers of the myometrium? a. Radial arteries b. Spiral arteries c. Straight arteries d. Arcuate arteries
radial arteries
What are the potential differential gynecologic diagnosis of urinary frequency?
Leiomyoma, leiomyosarcoma
During pregnancy the isthmus may be referred to as what?
Lower uterine segment
The total number of completed pregnancies that I've reached the age of viability
Parity
Something that grows off a stalk
Pedunculated
Contractions that move in a wave-like pattern to propel a substance
Peristalsis
Location of leiomyoma in which the tumor grows outward and distorts the Contour of the uterus
Subserosal
Who recommends the proper staging of ovarian carcinoma?
The international Federation of Gynecology and obstetrics FIGO
Following Menses how does the endometrium appear?
Thin package anticline
The boundaries of the female pelvis are considered to be from what?
Iliac crest to a group of muscles known as the pelvic diaphragm located at the base of the pelvis
What pelvic muscles are lateral and anterior to iliac crest?
Iliopsoas
False pelvis bilateral muscles located lateral to the uterus and anterior to the iliac crest
Iliopsoas muscle
The innominate bones consist of what?
Ilium, ischium, pubic symphysis
A vaginal anomaly in which the hymen has no opening, therefore resulting in an obstruction of the vagina
Imperforate hymen
Hypothalamus a vaginal anomaly in which the hymen has no opening resulting in an obstruction of the vagina
Imperforate hymen
How might a Cervical Carcinoma appear sonography?
In homogeneous, enlarged cervix or a focal Mass within the cervix
The distal segment of the fallopian tube
Infundibulum
What are the sonography findings of pyosalpinx and hematosalpinx?
Internal components and may appear echogenic or have a fluid fluid level
Internal branches of the common iliac arteries
Internal iliac arteries
The right and left uterine arteries are branches of what?
Internal iliac arteries
The superior portion of the cervix closest to the isthmus
Internal os
What hormone gives the surge to trigger ovulation
LH
The *two hormones produced by the anterior pituitary gland* that impact the menstrual cycle are:
LH and FSH
During which phase of the endometrial cycle would the endometrium yield the *three-line sign*?
Late proliferative
What are the potential differential gynecologic diagnosis of palpable adnexal Mass?
Leiomyoma, ovarian Mass, pelvic inflammatory disease
An elevated white blood cell count
Leukocytosis
Imaginary line that separates the true pelvis from the false pelvis
Linea terminalis
Menstrual bleeding
Menses
Fibromas are most often found in what aged women?
Middle-aged
Are ovarian fibromas associated with estrogen?
No
Days 15 through 28 what phase is over in endometrium in?
Ovary is in luteal and endometrium is in secretory
Leiomyosarcoma are more commonly found in what woman?
Perimenopausal or postmenopausal
Inflammation of the peritoneal lining
Peritonitis
What pelvic muscles are posterior?
Piriformis
Thecoma are most often found in whom?
Postmenopausal women, Meigs syndrome
Paired anterior abdominal muscles that extend from the xiphoid process of the sternum to the pubic bone
Rectus abdominis muscles
What age are dermoids commonly found in?
Reproductive age group but may also be found in the postmenopausal patients
Inflammation of the fallopian tubes
Salpingitis
Ovarian tumors that arise from The gonadal Ridges
Sex cord stromal tumors
What are the sonography findings of granulosa cell tumor?
Solid hypoechoic Mass, complex or partially cystic mass
Coiled arteries that Supply blood to the functional layer of the endometrium
Spiral arteries
Uterine radial artery branch that supplies blood to the basal layer of the endometrium
Straight arteries
Estrogen is produced throughout the menstrual cycle it is initially produced by what?
Theca internal cells of the secondary follicles during the first part of the menstrual cycle, during this phase estrogen initiates the proliferation and thickness of the endometrium by encouraging the growth and expansion of spiral arteries and glands within the functional layer of the endometrium, oxygen has many other important functions, regeneration of the endometrium after Menses and induction of salt and water retention, simulates contractile emotions within the uterine myometrium and Fallopian tubes, during second half dimensional cycle following ovulation progesterone is produced by the corpus luteum of the ovary, progesterone is responsible for maintaining the thickness of the endometrium and inducing its secretory activity as it is prepared for implantation
Benign ovarian sex cord stromal tumor that produces estrogen in older woman
Thecoma
In the secretory phase how does endometrium appear?
Thickened and academic measuring between 7 and 14 mm
The ovaries form and where and descend to the pelvis in utero?
Upper abdomen
What are the sonography findings of yolk Sac tumor?
Varying sonography appearances
Changes within the female that are caused by increased androgens, may lead to deepening of the voice in hirsutism
Virilization
Where is the ovum contained?
Within the cumulus oophorus
What term describes the echogenicity of a simple ovarian cyst?
a. anechoic
abnormally heavy and prolonged menstrual flow between periods is termed: a. menometrorrhagia b. menarche c. menorrhagia d. dysmenorrhea
a. menometrorrhagia
all of the following are clinical findings associated with leiomyoma except: a. myometrial cyst b. infertility c. palpable pelvic mass d. menorrhagia
a. myometrial cyst
precocious puberty is defined as the development of pubic hair, breasts, and the genitals before the age of: a. 13 b. 8 c. 5 d. 10
b. 8
what Doppler artifact occurs when the Doppler sampling rate is not high enough to display the Doppler shift frequency?
b. aliasing
the inner mucosal lining of the uterus is the: a. myometrium b. endometrium c. serosal layer d. perimetrium
b. endometrium
Anechoic fluid noted distending the uterus and cervix within a pediatric patient is termed: a. hydrocolpos b. hydrometrocolpos c. hydrometra d. hematometrocolpos
b. hydrometrocolpos
having the same echogenicity means:
b. isoechoic
what section of the uterus is also referred to as the lower uterine segment: a. cervix b. isthmus c. fundus d. cornu
b. isthmus
- small, solid, hypoechoic, unilateral tumors with calcifications - benign
brenner tumor aka transitional cell tumor
amernorrhea is defined as:
c. lack of menstrual flow
what leiomyoma location have an increased risk to undergo torsion? a. subserosal b. intracavity c. pedunulated d. submucosal
c. pedunulated
hairlike structures in the tube lumen
cilia shift
- considered most common cyst of 1st trimester pregnancy - often complex, thick walled - may resemble ectopic pregnancy
corpus luteum cyst
leiomyomas that project from a stalk are termed: a. submucosal b. intramural c. subserosal d. pedunculated
d. pedunculated
which of the following would be considered the more common uterine anomaly? a. bicornis univernus b. bicornus bicollis c. uterus didelphys d. septate uterus
d. septate uterus
if fertilization occurs menses begins when
day 1 of cycle
FSH causes what
follicle maturation
The hormone produced by the *trophoblastic cells* of the early placenta is:
hCG
the ovum is contained where
in the cumulus oophorus of dominant follicle
Hammock shaped pelvis muscle group located between the coccyx and pubis consisting of iliococcygeus, pubococcygeus and puborectalis
levator ani muscles
The pelvic muscle group that is located between the coccyx and the pubis is the: a. Levator ani muscles b. Rectus abdominis muscles c. Obturator internus muscles d. Piriformis muscle
levator ani muscles
second half of menstrual cycle
luteal phase
late proliferative phase is also known as
periovulatory phase
Pelvic bones, when visualized on sonography, will produce: a. Posterior shadowing b. Posterior enhancement c. Mirror image artifact d. Minimal enhancement
posterior shadowing
the fluid from the ruptured follicle most often will settle in the ...
rectouterine pouch (pouch of douglas)
pelvic muscles that can be sonographically identified include
rectus abdominis, iliopsoas, obturator internus, piriformis, levator ani, coccygeus
may be associated with endocrinologic abnormalities
secondary amenorrhea
occurs after ovulation and stimulated by progesterone
secretory phase
the testicular equivalent of an ovarian dysgerminoma is the ...
seminoma
most common malignancy in the ovary, bilateral, thicker septations, more projections
serous cystadenoma
what two thing comprise majority of neoplasms of the ovary
serous cystadenomas and cystic teratomas
sonographic appearance of fallopian tube carcinoma is what
solid mass within the adnexa
what sign from CD may be present in a torsed ovary
whirlpool sign
As a result of constant estrogen stimulation postmenopausal patients with granulosa cell tumors have approximately what percentage chance of developing endometrial carcinoma?
10 to 15
*Ovulation* typically *occurs* on day-of the menstrual cycle.
14
around what day is there a surge in LH
14
The normal pre-pubertal uterus has a cervix to uterus ratio of what?
2 to 1
The fingerlike extensions of the Fallopian tube are called: A. Fimbria B. Infundibulum C. Cilia D. Ampulla
A. Fimbria
A 55-year old patient presents to the sonography department with a history of pelvic pressure, abdominal swelling, and abnormal uterine bleeding. A pelvic sonogram reveals a large, multiloculated cystic mass with papillary projections. What is the most likely diagnosis? A. Serous cystadenocarcinoma B. Cystic teratoma C. Androblastoma D. Dysgerminoma
A. Serous cystadenocarcinoma
A change in *menstrual bleeding associated with lesions* within the uterus relates to:
AUB
In the early proliferative phase the endometrium appears how?
Academic and then typically measuring 4 mm on day 4 and 8 mm on day 8 of the cycle
Congenital malformations of the vagina can lead to what?
Accumulation of fluid within the female genital tract secondary to an obstruction
What are the clinical findings of ovarian torsion?
Acute unilateral abdominal or pelvic pain, nausea and vomiting
What are the potential differential gynecologic diagnosis of enlarged uterus?
Adenomyosis, endometrial carcinoma, leiomyoma, leiomyosarcoma
The size and shape of the uterus depends on what?
Age of patient, parity, presence of pathology or congenital anomalies
The typical version of the uterus where the uterine body tilts forward, forming a 90 degree angle with the cervix
Anteversion
The normal position of the uterus is considered to be what?
Anteversion or anteflexion
Major abdominal artery responsible for supplying the abdomen, pelvis, and lower extremities with oxygenated blood
Aorta
What are the clinical findings of a nabothian cyst?
Asymptomatic
Degeneration of a follicle
Atresia or ovarian follicle
Pus within the Fallopian tube is termed: A. Hematosalpinx B. Pyosalpinx C. Hydrosalpinx D. Hemosalpinx
B. Pyosalpinx
Another name for the endometriomas
Chocolate cysts
Hairlike projections within the fallopian tube
Cilia
The space of Retzius is located: a. Between the uterus and bladder b. Between the bladder and ilium c. Along the lateral aspect of the uterus d. Between the bladder and pubic bone
D. Between the bladder and pubic bone
The ovarian cysts that are most often bilateral and are associated with markedly elevated levels of hCG are the: A. Corpus luteum cysts B. Paraovarian cysts C. Granulosa cell cysts D. Theca lutein cysts
D. Theca lutein cysts
A small cyst within a large cyst
Daughter cyst
What are the sonography findings of adenomyosis?
Diffusely enlarged uterus, hypoechoic or academic areas adjacent to the endometrium, heterogeneous myometrium, myometrial cyst, ill-defined interface between myometrium and endometrium, thickening of the posterior myometrium
In the neonatal period what does the uterus look like?
Distinct endometrial echoes in the first week of life
Painful or difficult urination
Dysuria
Theca lutein cysts are found in the presence of what? What are common concurrent conditions?
Elevated levels of human chorionic gonadotropin in excess of 100,000,, gestational trophoblastic disease and ovarian hyperstimulation syndrome and multiple just stations
What are the clinical findings of yolk Sac tumor?
Elevation in serum AFP
The degeneration of the endometrium with advancing age most often seen in postmenopausal women
Endometrial atrophy
Cancer of the endometrium
Endometrial carcinoma
An *increase in the number of endometrial cells* is termed:
Endometrial hyperplasia
An increase in the number of endometrial cells
Endometrial hyperplasia
What are the potential differential gynecologic diagnosis of intermenstrual bleeding?
Endometrial polyp
Small nodules of hyperplastic endometrial tissue
Endometrial polyps
Benign, blood containing tumor that forms from the implementation of ectopic endometrial tissue, tumor associated with endometriosis
Endometrioma
Functional ectopic endometrial tissue located outside the uterus
Endometriosis
The *hormone of the pituitary gland that stimulates* follicular development of the ovary is:
FSH
Superior portion of the pelvis
False pelvis
What treatment will also result in the development of multiple, enlarged follicular cysts?
Fertility treatment
Adenomyosis is often present in the uterus Afflicted with what?
Fibroid tumors
the dominant follicle
Graafian follicle
A laboratory value that indicates the amount of red blood cells in blood
Hematocrit
Blood within the fallopian tube
Hematosalpinx
The inability to conceive a child after one year of unprotected intercourse
Infertility
The segment of the fallopian tube that lies within the uterine horn
Interstitial
A leiomyoma located within the uterine
Intracavitary
What kind of glands are the ovaries?
Intraperitoneal and endocrine
Area of the uterus between the Corpus and the cervix
Isthmus
Malignant ovarian tumor that metastasizes from the GI tract
Krukenberg tumor
What ovarian neoplasm is a result of metastases of a gastrointestinal cancer?
Krukenberg tumor
formula for ovarian volume
LENGTH X WIDTH X HEIGHT X 0.523
what artifact could be noted emanating from air or gas within the endometrium in a patient with endometritis?
!!! a. Ring-down
endovaginal transducers may be cleaned by submerging in a(n) ____-based solution:
!!! a. glutaraldehyde
which of the following laboratory tests may be used as a tumor marker for an ovarian dysgerminoma?
!!! a. lactate dehydrogenase
Ovulation typically occurs around what day?
14
Follicular cysts range in size from what?
3 to 8 cm
How many layers does the fallopian tube have and what are they?
3, outer serosa, middle muscular layer, inner mucosal layer
How many stages of ovarian carcinoma are there?
4
In the early proliferative phase what is endometrial thickness?
4 through 8 mm
The *measurement of the endometrium during the early* proliferative phase ranges from:
4 to 8 mm
In Periovulatory phase what is endometrial thickness?
6 through 10 mm
Which of the following is also referred to as a chocolate cyst? A. Endometrioma B. Endometroid C. Cystic Teratoma D. Androblastoma
A. Endometrioma
A focal mass of adenomyosis
Adenomyoma
Shadowing ovarian torsion is much more common in whom?
Adolescence
Failure of an organ to grow during embryologic development
Agenesis
A protein produced by the fetal yolk Sac, fetal gastrointestinal tract, and the fetal liver may also be produced by some malignant tumors
Alpha fetoprotein
Absence of menstrual period
Amenorrhea
The absence of menstrual period
Amenorrhea
The absence of menstrual.
Amenorrhea
The longest and most tortuous segment of the fallopian tube
Ampulla
*Ectopic endometrial tissue* within the uterus that *leads to AUB* is termed:
Andenomyosis
Sertoli leydig cell tumor AKA
Androblastoma
Peritoneal outpouching located between the bladder in the uterus AKA vesico uterine pouch
Anterior cul-de-sac
Peritoneal outpouching located between the bladder in the uterus AKA vesicouterine pouch
Anterior cul-de-sac
What are the potential differential gynecologic diagnosis of infertility?
Asherman syndrome, endometrial carcinoma, endometrial polyp, endometriosis, leiomyoma, pelvic inflammatory disease, polycystic ovarian disease
What are the potential differential gynecologic diagnosis of post dilation and postcurettage?
Asherman syndrome, endometritis, retained products of conception
What are the clinical findings of a fibroma?
Asymptomatic, Meigs syndrome
What are the clinical findings of a Brenner tumor?
Asymptomatic, Meigs syndrome, palpable mass or pain
The longest and most tortuous segment of the Fallopian tube is the: A. Fimbria B. Ampulla C. Isthmus D. Interstitial
B. Ampulla
What substance does hysterosalpingography utilize for the visualization of the uterine cavity and Fallopian tubes? A. Saline B. Radiographic contrast C. Water D. Betadine
B. Radiographic contrast
Which of the following is the most common malignancy of the ovary? A. Cystic teratoma B. Serous cystadenocarcinoma C. Krukenberg tumor D. Sertoli- Lyedig cell tumor
B. Serous cystadenocarcinoma
The ovarian cyst associated with gestational trophoblastic disease is the: A. Corpus luteum cyst B. Theca lutein cyst C. Dermoid cyst D. Paraovarian cyst
B. Theca lutein cyst
The non-functional outer layer of the endometrium
Basal layer
The follicular phase of the ovarian cycle is considered to begin on what day and last until what day?
Begins on day one and lasts until day 14 thus in effect with ovulation
What ligament extends from the lateral aspect of the uterus to the side walls of the pelvis and supports the uterus, tubes, ovaries?
Broad ligaments
The muscle located lateral to the ovaries is the: a. Iliopsoas muscle b. Rectus abdominis muscle c. Obturator internus muscle d. Piriformis muscle
C) Obturator internus muscle
Pelvic muscles appear: a. Echogenic b. Anechoic c. Hypoechoic d. Complex
C) hypoechoic
Hairlike projections within the fallopian tube are called: A. Interstita B. Fimbria C. Cilia D. Peristalsis
C. Cilia
After the Graafian follicle ruptures, the remaining structure is termed the: A. Graafian remnant B. Corpus albicans C. Corpus luteum D. Theca lutein cyst
C. Corpus luteum
Sonographically, which of the following would most likely be confused for a pedunculated fibroid tumor because of its solid appearing structure? A. Serous cystadenoma B. Mucinous cystadeoma C. Fibroma D. Theca lutein cyst
C. Fibroma
With what ovarian tumor is Meigs syndrome most likely associated? A. Dysgerminoma B. Cystic teratoma C. Fibroma D. Yolk sac tumor
C. Fibroma
The right ovarian vein drains directly into the: a. Right renal vein b. Aorta c. Inferior vena cava d. Common iliac vein
C. Inferior vena cava
The true pelvis is delineated from the false pelvis by the: a. Space of Retzius b. Adnexa c. Linea terminalis d. Iliac crest
C. Linea terminalis
An endometrioma most likely appears as a: A. Simple, anechoic mass with through transmission B. Complex mass with internal shadowing components C. Mostly cystic mass with low-level echoes D. Solid, hyper echoic shadowing mass
C. Mostly cystic mass with low-level echoes
What would be a predisposing condition that would increase the risk for suffering from ovarian torsion? A. Hirsutism B. Excessive exercise C. Ovarian mass D. Sonohysterography
C. Ovarian mass
elevated levels of what is a protein that is increased in women with ovarian cancer
CA- 125
A diagnostic modality that utilizes ionizing radiation to produce images of the human body in cross-sectional and reconstructed 3D formats
Computed tomography
Primary amenorrhea may be caused by what?
Congenital abnormalities or congenital obstruction such as imperforate hymen
The remaining structure of the corpus luteum after it is deterioration
Corpus albicans
The remaining structure of the corpus luteum after its deterioration
Corpus albicans
Which structure remains after the *corpus luteum has regressed*?
Corpus albicans
After the graafian follicle is ruptured its structure is converted into what?
Corpus luteum
Temporary endocrine gland that results from the rupture of the graafian follicle after ovulation
Corpus luteum
The *temporary endocrine gland* that results from the rupture of the Graafian follicle is the:
Corpus luteum
The temporary endocrine gland that results from the rupture of the graafian follicle after ovulation
Corpus luteum
physiologic cyst that develops after ovulation has occurred
Corpus luteum cyst
Monthly symptoms of menstruation without bleeding
Cryptomenorrhea
Structure that contains the developing oocyte
Cumulus oophorus
The structure noted within the *Graafian follicle containing the developing ovum* is the:
Cumulus oophorus
The structure that contains the developing oocyte
Cumulus oophorus
Benign ovarian mass that is composed of the three germ cell layers AKA dermoid cyst
Cystic teratoma
The bilateral muscles that are located posterior to and extend from the sacrum to the femoral greater trochanter are the: a. Levator ani muscles b. Rectus abdominis muscles c. Obturator internus muscles d. Piriformis muscles
D) Piriformis muscles
The segment of Fallopian tube where fertilization typically occurs is the: A. Cornu B. Fimbria C. Interstitial D. Ampulla
D. Ampulla
Which of the following is a tumor of ectopic endometrial tissue? A. Brenner tumor B. Cystic teratoma C. Yolk-sac tumor D. Endometrioma
D. Endometrioma
The most distal part of the Fallopian tube is the: A. Cornu B. Ampulla C. Interstitial D. Infundibulum
D. Infundibulum
The short and narrow segment of the Fallopian tube distal to the interstitial segment is the: A. Ampulla B. Fimbria C. Infundibulum D. Isthmus
D. Isthmus
The malignant ovarian mass that is associated with pseudomyxoma peritonei is the: A. Dysgerminoma B. Sertoli-Leidig cell tumor C. Serous cystadenocarcinoma D. Mucinous cystandenocarcinoma
D. Mucinous cystandenocarcinoma
Another name for a cystic teratoma
Dermoid cyst
Mass of hair within a cystic teratoma
Dermoid mesh
Part of a dermoid tumor that contains various tissues and may produce the posterior shadowing during a sonography exam
Dermoid plug
Describe the condition of stage 4 ovarian carcinoma
Distant Mets behind the peritoneal cavity
What are the sonography findings of vaginal obstructions?
Distention of the uterus, vagina, or both with anechoic or complex fluid
Abnormal uterine bleeding is usually caused by what?
Endocrine abnormalities or lesions within the uterus
Area that lies between the two layers of the endometrium AKA uterine cavity
Endometrial cavity
What are the potential differential gynecologic diagnosis of tamoxifen therapy?
Endometrial hyperplasia
What is the most likely explanation for abnormal uterine bleeding with an endometrial measurement of <5 mm in a postmenopausal patient?
Endometrial hyperplasia
What are the potential differential gynecologic diagnosis of menorrhagia?
Endometrial hyperplasia, endometriosis, leiomyoma, leiomyosarcoma
An important differentiation should be made between endometriosis and adenomyosis. Discuss the differences.
Endometriosis tend to be younger and have fertility problems while adenomyosis are often older and multiparous
The hormone released by the ovary during the proliferative phase then initiates the proliferation and thickening of the endometrium
Estrogen
Which hormone released by the ovary during proliferative phase *stimulates endometrial thickening*?
Estrogen
The ovary produces what two hormones during the menstrual cycle?
Estrogen and progesterone
What are the potential differential gynecologic diagnosis of lost intrauterine contraceptive device?
Expelled intrauterine device, perforated myometrium / uterus
The inferior portion of the cervix in close contact with the vagina
External OS
External branches of the common iliac arteries
External iliac arteries
The vagina is a tubular organ which extends from where to where?
External os of the cervix to the external genitalia
During the follicular phase the anterior pituitary gland secretes what which initiates the follicular development of the ovary many follicles are produced by the ovary?
FSH
during the follicular phase, the ant pit gland produces what
FSH
two main hormones produced by the anterior pituitary gland
FSH and LH
ovaries are stimulated by what
FSH produced by anterior pituitary gland
A patient presents with abnormal uterine bleeding. What is the most common cause?
Fibroid tumors invading the endometrial cavity
An ovarian sex cord stromal tumor found in middle aged women
Fibroma
The finger-like extensions of the fallopian tube located on the infundibulum
Fimbria
The first day of the menstrual cycle is said to occur on what?
First day of bleeding
Small round group of cells
Follicle
Should the graafian follicle failed to ovulate it could continue to enlarge and result in what?
Follicular cyst
The *first phase of the ovarian cycle* is the:
Follicular phase
The first phase of ovarian cycle
Follicular phase
The ovarian cycle consists of what two phases?
Follicular phase and luteal phase
The most Superior and widest portion of the uterus
Fundus
The uterus can be divided into what four major divisions?
Fundus, Corpus, Isthmus, cervix
I type of neoplasm derived from germ cells of the gonads, may also be found outside of the reproductive tract
Germ cell tumor
A disease associated with an abnormal proliferation of the trophoblast cells during pregnancy AKA molar pregnancy
Gestational trophoblastic disease
Blood accumulation within the vagina
Hematocolpos
What are the sonography findings of uterine leiomyoma?
Hypoechoic Mass within the uterus, posterior shadowing from the mass, degenerating fibroids may have calcifications or cystic components, multiple fibroids appear as an enlarged irregular shaped diffusely heterogeneous uterus
What are the sonography findings of a thecoma?
Hypoechoic solid Mass with posterior attenuation, no posterior enhancement, if large May mimic a pedunculated leiomyoma
What are the potential differential gynecologic diagnosis of virilization?
Sertoli leydig cell tumor, ovarian carcinoma
The cervix is marked Superiorly by what and inferiorly by what?
Superior is the internal os and inferior is the external os
What are the clinical findings of serous and mucinous cystadenocarcinoma?
Weight loss, pelvic pressure and swelling, abnormal vaginal bleeding, GI symptoms, acute abdominal pain associated with torsion or rupture, elevated ca-125
Fluid noted posterior to the uterus would most likely be located within the: a. Space of Retzius b. Pouch of Douglas c. Anterior cul-de-sac d. Adnexa
b. pouch of douglas
common suspect of AUB
fibroid tumors, leiomyomas
At what day of the menstrual cycle does ovulation occur as the dominant follicle ruptures releasing the mature ovum and a small amount of follicular fluid into the peritoneal cavity?
14
fertilization occurs on what day
15
Excess fluid in the peritoneal cavity
Ascites
What are the potential differential gynecologic diagnosis of Meigs syndrome?
Brenner tumor fibroma thecoma
Small benign ovarian tumors
Brenner tumors
What is the most common female malignancy in women younger than 50?
Cervical Carcinoma
The rigid region of the uterus located between the Isthmus and vagina
Cervix
What are the clinical findings of dysgerminoma?
Children - precocious puberty, elevated serum lactate dehydrogenase, possible elevated serum HCG
Abdominal aortic bifurcation vessels
Common iliac arteries
What is the largest part of the uterus?
Corpus or body
*FSH is produced* by the:
Anterior pituitary gland
*LH is produced* by the:
Anterior pituitary gland
The anterior segment of the pituitary gland which is responsible for releasing follicle-stimulating hormone and luteinizing hormone during the menstrual cycle
Anterior pituitary gland
Where can endometriomas be located?
Anywhere outside of the endometrial cavity including on any other pelvic organs such as the bladder and bowel but are more commonly found in the ovary
Peripheral arteries of the uterus that lie at the edge of the myometrium
Arcuate arteries
What are the branches of the uterine artery?
Arcuate artery then to radial arteries then to straight arteries and spiral
A collection of abdominal fluid within the peritoneal cavity
Ascites
What are the potential differential gynecologic diagnosis of abdominal distention?
Ascites, leiomyoma, ovarian hyperstimulation syndrome, ovarian malignancy
What are the potential differential gynecologic diagnosis of hypomenorrhea?
Asherman syndrome
What is the potential differential gynecologic diagnosis for amenorrhea?
Asherman syndrome, polycystic ovarian disease
What are the clinical findings of a Gartner duct cyst?
Asymptomatic
What are the clinical findings of serous cystadenoma?
Asymptomatic
What are the clinical findings of a follicular cyst?
Asymptomatic, pain associated with Hemorrhage and enlargement of cyst
Painful sexual intercourse
Dyspareunia
A pregnancy located outside of the endometrial cavity of the uterus
Ectopic pregnancy
Hematocrit is laboratory value that should be evaluated in cases of suspected what?
Ectopic pregnancy
Secondary amenorrhea may be associated with what?
Endocrinologic abnormalities or pregnancy, secondary amenorrhea that is not associated with pregnancy is characteristically diagnosed and the postmenarcheal woman who has had at least 12 months without a menstrual cycle
Yolk Sac tumor AKA?
Endodermal sinus tumor
A catheter placed into the urinary bladder via the urethra that is used to drain urine, it can also be claimed and used to temporarily distend the bladder for a pelvic sonography
Foley catheter
Endovaginal transducers should be soaked in what?
Glutaraldehyde based solution
The *hormone produced by the hypothalamus* that *controls* the release of the hormones for menstruation by the anterior pituitary gland is:
GnRH
The *dominant follicle* prior to ovulation is termed the:
Graafian follicle
How is the volume of the ovary determined?
Length X width X height X 0.5233.2
The hormone of the anterior pituitary gland that surges around day 14 of the menstrual cycle resulting in ovulation
Luteinizing hormone
What are the sonography findings of a krukenberg tumor?
Bilateral, smooth walled hypoechoic ovarian Mass, may have ascites
The pituitary gland AKA? Located where? Responsible for?
Master gland, endocrine gland located within the brain that consists of an anterior and posterior lobe, anterior lobe responsible for the release of two hormones follicle-stimulating hormone and luteinizing hormone, FSH causes the development of multiple follicles in the ovaries, LH surges around day 14 of the menstrual cycle which results in ovulation
A diagnostic imaging modality that utilizes the administration of radionuclides into the human body for an analysis of the function of organs, or for the treatment of various abnormalities
Nuclear medicine
What are the clinical findings of a cystic teratoma?
Often asymptomatic, if torsion or rupture occurs the patient may present with acute pelvic pain
What are the ovaries consist of?
Outer cortex and inner medulla. Medulla contains ovarian vasculature and lymphocytes, cortex involves the mass of the ovary and is the site of the oogenesis
The suspensory ligament of the ovary contains what?
Ovarian artery, ovarian vein, lymphatics, ovarian nerves
The surgical removal of an ovarian cyst
Ovarian cystectomy
What are the potential differential gynecologic diagnosis of elevated serum human chorionic gonadotropin in non gravid?
Ovarian dysgerminoma
Which of the following would *not be a cause of AUB (abnormal uterine bleeding)*?
Ovarian torsion
Following Menses the ovary is in what phase while the endometrium is in what phase?
Ovary is in the follicular phase while the endometrium is in the proliferative phase
What are the sonography findings of dysgerminoma?
Ovoid, solid echogenic Mass on the ovary, may contain some cystic components
A tumor marker in the blood that can indicate certain types of cancer such as cancer of the ovary, endometrium, breast, gastrointestinal tract, and lungs, stands for cancer antigen 125
CA - 125
The anterior cul-de-sac is also referred to as the: a. Space of Retzius b. Rectouterine pouch c. Pouch of Douglas d. Vesicouterine pouch
D. Vesicouterine pouch
What are the sonography appearance of a mucinous cystadenoma?
Large predominantly anechoic lesion that contains septations and or papillary projections, may contain some recognizable internal debris, not often bilateral
What are the sonography findings of serous cystadenoma?
Large, bilateral, predominantly anechoic lesion that contain septations and or papillary projections
Paired pelvic muscles located lateral to the ovaries
Obturator internus muscle
An abnormality that results from the ovary twisting on its mesenteric connection, consequently cutting off the blood supply to the ovary
Ovarian torsion
Together what comprise most neoplasms of the ovary?
Serous cystadenoma and cystic teratoma
upon sonographic evaluation of a patient complaining of abnormal distention, you visualize a large, hypoechoic mass distorting the anterior border of the uterus. what is the most likely location of the mass? a. intramural b. subserosal c. submucosal d. intracavity pedunculated
b. subserosal
pseudoprecocious puberty may be associated with all of the following except: a. ovarian tumor b. adrenal tumor c. liver tumor d. brain tumor
d. brain tumor
paracolic gutters
extend alongside the ascending and descending colon on both sides of the abdomen
may be referred to as oviducts, uterine tubes, or saplings
fallopian tubes
- sex cord stromal not associated with estrogen production - benign masses - middle aged women - hypoechoic, solid, post attenuation
fibroma
What is an estrogen producing tumor that is considered to be the most common estrogenic tumor?
Granulosa cell tumor
What are some laboratory values that may warrant a pelvic sonogram?
Human chorionic gonadotropin, hematocrit, white blood cell count
What is the typical sonographic *appearance of the endometrium during the secretory phase*?
Hyperchoic and thick
What is treatment for adenomyosis?
Hysterectomy or hormone therapy
What are the potential differential gynecologic diagnosis of elevated ca-125?
Endometriosis, leiomyoma, ovarian carcinoma, pelvic inflammatory disease
What are the potential differential gynecologic diagnosis of fever?
Endometritis, pelvic inflammatory disease
A means of looking inside the human body by utilizing an endoscope
Endoscopy
Excessive hair growth in women in areas where hair growth is normally negligible
Hirsutism
The dilation of the renal collecting system resulting from the obstruction of the flow of urine from the kidneys to the bladder AKA pelvocaliectasis
Hydronephrosis
Decreased or scant menstrual flow
Hypomenorrhagia
A common form of birth control in which a small device is placed within the endometrium to prevent the implementation of a pregnancy AKA intrauterine contraceptive device
Intrauterine device
What are the potential differential gynecologic diagnosis of dysuria?
Leiomyoma, leiomyosarcoma
A diagnostic modality that utilizes electromagnetic radiation to produce images of the human body in cross-sectional and reconstructed 3D formats
Magnetic resonance imaging
Heavy and prolonged bleeding between periods
Menometrorrhagia
What are the potential differential gynecologic diagnosis of acute pelvic pain?
Ovarian mucinous cystadenocarcinoma, ovarian mucinous cystadenoma, ovarian serous cystadenocarcinoma, ovarian serous cystadenoma, ovarian torsion, pelvic inflammatory disease, ruptured ovarian hemorrhagic cyst, perforated intrauterine contraceptive device
which of the following is best defined as difficult or painful menstruation? a. dysmenorrhea b. dyspareunia c. dysuria d. menorrhagia
a. dysmenorrhea
all of the following statements are true of endovaginal imaging except: a. endovaginal imaging requires a full urinary bladder b. endovaginal imaging leads to reduced waiting time for the patient and quicker medical management c. endovaginal imaging offers improved resolution of the endometrium, uterus, and ovaries, especially in the obese patient d. endovaginal imaging is contraindicated for pediatric patients, and for those with an intact hymen
a. endovaginal imaging requires a full urinary bladder
which of the following most often leads to an elevation of CA-125? a. ovarian carcinoma b. Fitz-Hugh-Curtis syndrome c. ovarian torsion d. ovarian hyperstimulation syndrome
a. ovarian carcinoma
which of the following would most likely be associated with hirsutism? a. polycystic ovary syndrome b. Meigs syndrome c. adenomyosis d. adenomyomatosis
a. polycystic ovary syndrome
precocious puberty is best defined as:
a. pubertal development before the age of 8
which of the following is defined as excessive hair growth in women in areas where hair growth is normally negligible?
b. hirsutism
which of the following would be best defined as abnormally heavy menstrual flow? a. menometrorrhagia b. menorrhgia c. metrorrhagia d. hypomenorrhea
b. menorrhagia
which of the following is best described as an artifact that is produced by a strong reflector and results in a copy of the anatomy being placed deeper than the correct location?
b. mirror image
which of the following diagnostic tests is used to evaluate emitted radiation from the patient to assess the function of organs? a. magnetic resonance imaging b. nuclear medicine c. radiography d. computed tomography
b. nuclear medicine
malignant ovarian tumors may leak mucinous material, and this condition is known as:
b. pseudomyxoma peritonei
which of the following definitions best describes the term adnexa? a. the area posterior to the uterus, between the uterus and rectum b. the area located posterior to the broad ligaments and adjacent to the uterus c. the area anterior to the uterus, between the uterus and urinary bladder d. the area lateral to the iliac crest and posterior to the pubic symphysis
b. the area located posterior to the broad ligaments and adjacent to the uterus
all of the following are associated with acute pelvic pain except: a. pelvic inflammatory disease b. ruptured ovarian hemorrhagic cyst c. perforated intrauterine contraceptive device d. Asherman syndrome
d. Asherman syndrome
what term relates to the number of pregnancies a patient has had? a. para b. menarche c. menorrhagia d. gravida
d. gravida
which of the following would be best defined as regularly timed menses but light flow? a. menometrorrhagia b. menorrhgia c. metrorrhagia d. hypomenorrhea
d. hypomenorrhea
which of the following is best defined as intermenstrual bleeding? a. dysmenorrhea b. menorrhagia c. menometrorrhagia d. metrorrhagia
d. metrorrhagia
leukocytosis would most likely be associated with: a. multiple degenerating fibroids b. ovarian teratoma c. adenomyosis d. pelvic inflammatory disease
d. pelvic inflammatory disease
the best way to communicate with a patient who speaks a language other than your own is to?
d. use a trained medical interpreter
Painful or difficult urination
dysuria
Anterior cul-de-sac
peritoneal outpouching located between the bladder and the uterus; also referred to as the vesicouterine pouch
occurs after menstruation and lasts until ovulation
proliferative phase
fluid accumulation within the uterus and vagina
Hydrometrocolpos
What ovarian timor will most likely have a moth-eaten appearance on sonography? A. Cystic teratoma B. Serous cystadenocarcinoma C. Krukenberg tumor D. Sertoli- Leydig cell tumor
C. Krukenberg tumor
Which of the following is the correct formula for calculating ovarian volume? A. L x W x H x 0.6243 B. L x W x H x 0.3899 C. L x W x H x 0.5233 D. Ovarian volume cannot be calculated
C. L x W x H x 0.5233
Pelvic muscle located posteriorly within the pelvis that helps support the sacrum
Coccygeus
During a pelvic sonogram you visualize a small cyst located adjacent to the ovary. What is the most likely etiology of this cyst? A. Dermoid Cyst B. Ovarian cystadenoma C. Endometrioma D. Paraovarian Cyst
D. Paraovarian Cyst
What structure within the female pelvis lies posterior to the urinary bladder and anterior to the rectum? a. Broad ligament b. Rectus abdominus muscle c. Space of Retzius d. Uterus
D. Uterus
A patient with an ovarian mass presents with an elevated serum AFP. Which of the following would be most likely? A. Ovarian fibroma B. Ovarian thecoma C. Cystic teratoma D. Yolk-sac tumor
D. Yolk-sac tumor
Intramural for pedunculated fibroids that tourists what are the clinical findings?
Acute, localized pelvic pain
Hormone of the anterior pituitary gland that causes the development of multiple follicles on the ovaries
Follicle-stimulating hormone
A benign cyst located within the vagina
Gartner duct cyst
What are the sonography findings of a fibroma?
Hypoechoic solid Mass with posterior enhancement, no posterior enhancement, if large mimic a pedunculated leiomyoma